Why isn't {0}^{3}+{0}^{3}={0}^{3} a proof for Fermat's Last Theorem?

  • MHB
  • Thread starter Angel11
  • Start date
  • Tags
    Theorem
In summary, the equation {0}^{3}+{0}^{3}={0}^{3} cannot be used as a proof for Fermat's Last Theorem because it does not follow the necessary conditions set by the theorem. The theorem states that for any positive integer value of n greater than 2, the equation a^{n}+b^{n}=c^{n} has no integer solutions. However, in the case of {0}^{3}+{0}^{3}={0}^{3}, all the values of a, b, and c are equal to 0, which does not satisfy the condition of having different nonzero integer values. Therefore, this equation cannot be considered as a
  • #1
Angel11
11
0
Hello, It is me again.So i was watching some math videos and i came across Fermat's Last Theorem which was very intersting.But i was confused because i wondered for a second and sayed "well if A,B and C are equal then they could be 0 to prove it" but at the same time i thought "well if it works something like the pythagorean theorem then that would be impossible because if a triangle has 3 sides with the length of 0 then there would be nothing" BUT again i also thought "But Fermat's Last Theorem doesn't say anything about a right triangle or any triangle it is just the formula" So my question is:Why isn't {0}^{3}+{0}^{3}={0}^{3} proof (or on any other power with n>2)
 
Mathematics news on Phys.org
  • #2
I've moved this thread from Differential Equations to Number Theory as that's a better fit.

From Wikipedia:

In number theory, Fermat's Last Theorem (sometimes called Fermat's conjecture, especially in older texts) states that no three positive integers $a$, $b$, and $c$ satisfy the equation $a^n+b^n=c^n$ for any integer value of $n$ greater than 2. The cases $n=1$ and $n=2$ have been known to have infinitely many solutions since antiquity.
 
  • #3
oh i didn't realize the "positive number" how stupid of me. Also thanks for moving the thread to number theory. I put it hear because i didn't know where to put it and also thank you for replying
 
  • #4
Angel1 said:
oh i didn't realize the "positive number" how stupid of me.

I don't think there's anything "stupid" about investigating theorems. It can be easy to miss details, and so asking about it is smart. :D

Angel1 said:
Also thanks for moving the thread to number theory. I put it hear because i didn't know where to put it and also thank you for replying

In the future, if you are unsure about where to post a thread, just make your best guess (as you did for this thread), and then use the post reporting feature to call the thread to the attention of the staff.

To do so, look for the http://mathhelpboards.com/images/mhb/buttons/report-40b.png icon beneath the post, and click that and you will be presented with a form to enter the reason you're reporting the post. Once you enter the reason then submit the form.

When you report the post, just indicate that you are unsure about whether it's posted in the best forum, and someone on staff will be happy to move the thread if needed. (Yes)
 

What is Fermat's Last Theorem?

Fermat's Last Theorem is a mathematical conjecture proposed by French mathematician Pierre de Fermat in the 17th century. It states that no three positive integers a, b, and c can satisfy the equation an + bn = cn for any integer value of n greater than 2.

Has Fermat's Last Theorem been proven?

Yes, Fermat's Last Theorem was finally proven in 1994 by British mathematician Andrew Wiles, after over 300 years of attempts by mathematicians to solve it. Wiles' proof was based on advanced mathematical techniques, including modular forms and elliptic curves.

Why is Fermat's Last Theorem significant?

Fermat's Last Theorem is significant because it was one of the most famous unsolved problems in mathematics for centuries, and its proof required the development of new mathematical techniques and concepts. It also has connections to other areas of mathematics, such as number theory and algebraic geometry.

Can Fermat's Last Theorem be generalized for higher powers?

Yes, Fermat's Last Theorem can be generalized for higher powers. This is known as the generalized Fermat's Last Theorem, and it states that an + bn = cn has no integer solutions when n is a positive integer greater than 2, and a, b, and c are distinct coprime integers.

What are some applications of Fermat's Last Theorem?

Fermat's Last Theorem has applications in various areas of mathematics, including number theory, algebraic geometry, and mathematical physics. It has also inspired further research and developments in other mathematical concepts and theories.

Similar threads

Replies
15
Views
1K
Replies
1
Views
769
Replies
12
Views
2K
Replies
2
Views
1K
Replies
13
Views
1K
Replies
2
Views
1K
  • General Math
Replies
6
Views
1K
Replies
12
Views
1K
Replies
13
Views
1K
Back
Top